If a gymnast sitting on a rotating stool, with his arms outstretched suddenly lowers his arm

Applied Mechanics Mcqs

A.the angular velocity will decrease
B.his moment of inertia will decrease
C.the angular velocity will remain constant
D.the angular momentum will increase

Leave a Reply

Your email address will not be published. Required fields are marked *